Fix subscripting (r_12 -> r_{12}) in Serway and Jewett v8's 25.21.
[course.git] / latex / problems / Serway_and_Jewett_8 / problem25.21.tex
1 \begin{problem*}{25.21}
2 Two particles each with charge $+2.00\U{$\mu$C}$ are located on the
3 $x$ axis.  One is at $x=1.00\U{m}$, and the other is at
4 $x=-1.00\U{m}$.  \Part{a} Determine the electric potential on the $y$
5 axis at $y=0.500\U{m}$.  \Part{b} Calculate the change in electric
6 potential energy of the system as a third charged particle of
7 $-3.00\U{$\mu$C}$ is brought in from infinitely far away to a position
8 on the $y$ axis at $y=0.500\U{m}$.
9 \end{problem*}
10
11 \begin{solution}
12 \Part{a}
13 \begin{equation}
14   V_3 = k_e \frac{q_1}{r_{13}} + k_e \frac{q_2}{r_{12}}
15     = 2 k_e \frac{2\U{$\mu$C}}{\sqrt{(1.00\U{m})^2 + (0.500\U{m})^2}}
16     = \ans{32.2\U{kV}}
17 \end{equation}
18
19 \Part{b}
20 \begin{equation}
21   \Delta U = q_3 V_3 - q_3 V_\infty = q_3 V_3 = \ans{-96.5\U{mJ}}
22 \end{equation}
23 \end{solution}